Movimiento oscilatorio no SHM

ingrese la descripción de la imagen aquí

Cómo resolver este tipo de preguntas, donde | F | X 2 ? ¿Cómo encontrar cosas relacionadas con el período de tiempo y el tipo de velocidad con el movimiento oscilatorio?

metro d 2 X d t 2 = F = d tu d X = 3 k X | X | .

pero despues de esto

metro d 2 X d t 2 = 3 k X | X | .

¿Cuál es la solución general de esta EDO? creo que le daría la X en términos de t y podría obtener un período de tiempo a partir de él.

Pero aún así, si alguien puede darme una solución a este DE/
¿De dónde has sacado esto? @Mr.007

Respuestas (3)

Este tipo de preguntas de proporcionalidad a menudo se responden mejor con un análisis dimensional. Quieres saber una forma de cantidad con las unidades de tiempo en términos de lo que tienes.

tienes una cantidad k con unidades Energía Distancia 3 = Masa Distancia × Tiempo 2 . tambien tienes la masa metro (unidades de masa) y amplitud a (unidades de distancia). La única forma de combinar estas cantidades para obtener una respuesta con las unidades de tiempo es si la expresión es un número puro veces metro a k . Así que esto te dice cómo debe escalar el período con respecto a las constantes dimensionales.

Ok. ¡Fue bueno para este problema en particular! +1 gracias.
Solo para agregar a zkf, el teorema π de Buckingham es un método formal para calcular conjuntos de parámetros adimensionales a partir de las variables.
@zkf Buena respuesta. Bienvenido al sitio!

El análisis dimensional en la respuesta de zkf resuelve completamente el ejercicio. No obstante, es posible dar una fórmula cerrada para el período

T   =   4   metro 2 k 0 a d X a 3 X 3   = X = a tu   4   metro 2 k a 0 1 d tu 1 tu 3 .

¿Puedes ver por qué? Como era de esperar, esto solo confirma la respuesta de zkf.

¡Maldita sea, estaba escribiendo esto! :P El único bit adicional que hice (bueno, en realidad Mathematica lo hizo por mí): la integral resulta ser π Γ ( 4 3 ) / Γ ( 5 6 ) 1.40218 .

zkf le brinda suficiente para responder esta pregunta, pero me gustaría hacer algunos puntos adicionales:

  • La operación de valor absoluto en el potencial hace que este sea un problema no lineal, que generalmente es bastante difícil de tratar. Me impacienté esperando que Mathematica presentara una solución de forma cerrada para X ( t ) , por lo que probablemente no haya uno. Esta es la situación típica en física y en este caso hay que recurrir a un cálculo numérico.

  • Dado que este es un problema unidimensional, aún puede resolverlo utilizando la conservación de la energía (solo necesita una cantidad conservada para resolver una sola partícula en un problema 1D). Si escribes la conservación de la energía para este sistema:

1 2 metro X ˙ 2 + k | X | 3 = mi = constante ,

puedes reorganizar esto un poco en

d t = d X cosas que involucran x .

La integración le da la respuesta que QMechanic publicó hace un momento . :)